Difference between revisions of "1995 AHSME Problems/Problem 21"

m (fmt/tex/wik)
(See also)
Line 25: Line 25:
  
 
==See also==
 
==See also==
{{Old AMC12 box|year=1995|num-b=19|num-a=21}}
+
{{AHSME box|year=1995|num-b=19|num-a=21}}
  
 
[[Category:Introductory Geometry Problems]]
 
[[Category:Introductory Geometry Problems]]

Revision as of 08:54, 17 April 2008

Problem

Two nonadjacent vertices of a rectangle are $(4,3)$ and $(-4,-3)$, and the coordinates of the other two vertices are integers. The number of such rectangles is

$\mathrm{(A) \ 1 } \qquad \mathrm{(B) \ 2 } \qquad \mathrm{(C) \ 3 } \qquad \mathrm{(D) \ 4 } \qquad \mathrm{(E) \ 5 }$

Solution

The distance between $(4,3)$ and $(-4,-3)$ is $\sqrt{6^2+8^2}=10$. Therefore, if you circumscribe a circle around the rectangle, it has a center of $(0,0)$ with a radius of $10/2=5$. There are three cases:

  • Case 1: The point "above" the given diagonal is $(4,-3)$.

Then the point "below" the given diagonal is $(-4,3)$.


  • Case 2: The point "above" the given diagonal is $(0,5)$.

Then the point "below" the given diagonal is $(0,-5)$.


  • Case 3: The point "above" the given diagonal is $(-5,0)$.

Then the point "below" the given diagonal is $(5,0)$.


We have only three cases since there are $8$ lattice points on the circle. $\Rightarrow \mathrm{(C)}$

See also

1995 AHSME (ProblemsAnswer KeyResources)
Preceded by
Problem 19
Followed by
Problem 21
1 2 3 4 5 6 7 8 9 10 11 12 13 14 15 16 17 18 19 20 21 22 23 24 25 26 27 28 29 30
All AHSME Problems and Solutions